Skip to main content
Log in

On Explicit \(L^2\)-Convergence Rate Estimate for Underdamped Langevin Dynamics

  • Published:
Archive for Rational Mechanics and Analysis Aims and scope Submit manuscript

Abstract

We provide a refined explicit estimate of the exponential decay rate of underdamped Langevin dynamics in the \(L^2\) distance, based on a framework developed in Albritton et al. (Variational methods for the kinetic Fokker–Planck equation, arXiv arXiv:1902.04037, 2019). To achieve this, we first prove a Poincaré-type inequality with a Gibbs measure in space and a Gaussian measure in momentum. Our estimate provides a more explicit and simpler expression of the decay rate; moreover, when the potential is convex with a Poincaré constant \(m \ll 1\), our estimate shows the decay rate of \(O(\sqrt{m})\) after optimizing the choice of the friction coefficient, which is much faster than m for the overdamped Langevin dynamics.

This is a preview of subscription content, log in via an institution to check access.

Access this article

Price excludes VAT (USA)
Tax calculation will be finalised during checkout.

Instant access to the full article PDF.

Similar content being viewed by others

Data Availability Statement

This manuscript has no associated data.

References

  1. Albritton, D., Armstrong, S., Mourrat, J.-C., Novack, M.: Variational methods for the kinetic Fokker–Planck equation, arXiv preprint arXiv:1902.04037, 2019

  2. Andrieu, C., Durmus, A., Nüsken, N., Roussel, J.: Hypocoercivity of piecewise deterministic Markov process-Monte Carlo. Ann. Appl. Probab. 31(5), 2478–2517, 2021

    Article  MathSciNet  MATH  Google Scholar 

  3. Armstrong, S.: Answer to “Elliptic regularity with Gibbs measure satisfying Bakry–Emery condition”, MathOverflow. https://mathoverflow.net/q/335599 (version: 2019-07-06)

  4. Bakry, D., Cattiaux, P., Guillin, A.: Rate of convergence for ergodic continuous Markov processes: Lyapunov versus Poincaré. J. Funct. Anal. 254(3), 727–759, 2008

    Article  MathSciNet  MATH  Google Scholar 

  5. Bakry, D., Émery, M.: Diffusions hypercontractives, Séminaire de Probabilités XIX 1983/84, Springer, 177–206, 1985

  6. Bakry, D., Gentil, I., Ledoux, M.: Analysis and Geometry of Markov Diffusion Operators, Springer, Cham, 2014

  7. Baudoin, F.: Wasserstein contraction properties for hypoelliptic diffusions, arXiv:1602.04177 [math], 2016

  8. Baudoin, F.: Bakry–émery meet Villani. J. Funct. Anal. 273(7), 2275–2291, 2017

    Article  MathSciNet  MATH  Google Scholar 

  9. Baudoin, F., Gordina, M., Herzog, D.P.: Gamma calculus beyond Villani and explicit convergence estimates for Langevin dynamics with singular potentials. Arch. Rational Mech. Anal. 241(2), 765–804, 2021

    Article  ADS  MathSciNet  MATH  Google Scholar 

  10. Bernard, É., Fathi, M., Levitt, A., Stoltz, G.: Hypocoercivity with Schur complements. Annales Henri Lebesgue 5, 523–557, 2022

    Article  MathSciNet  MATH  Google Scholar 

  11. Bogovskii, M.E.: Solution of the First Boundary Value Problem for the Equation of Continuity of an Incompressible Medium, Doklady Akademii Nauk, vol. 248, Russian Academy of Sciences, 1037–1040, 1979

  12. Camrud, E., Herzog, D.P., Stoltz, G., Gordina, M.: Weighted \(L^2\)-contractivity of Langevin dynamics with singular potentials. Nonlinearity 35(2), 998, 2021

    Article  ADS  MATH  Google Scholar 

  13. Cattiaux, P., Guillin, A., Monmarché, P., Zhang, C.: Entropic multipliers method for Langevin diffusion and weighted log Sobolev inequalities. J. Funct. Anal. 277(11), 108288, 2019

    Article  MathSciNet  MATH  Google Scholar 

  14. Conrad, F., Grothaus, M.: Construction, ergodicity and rate of convergence of \({N}\)-particle Langevin dynamics with singular potentials. J. Evol. Equ. 10(3), 623–662, 2010

    Article  MathSciNet  MATH  Google Scholar 

  15. Cooke, B., Herzog, D.P., Mattingly, J.C., McKinley, S.A., Schmidler, S.C.: Geometric ergodicity of two-dimensional Hamiltonian systems with a Lennard–Jones-like repulsive potential. Commun. Math. Sci. 15(7), 1987–2025, 2017

    Article  MathSciNet  MATH  Google Scholar 

  16. Dalalyan, A.S., Riou-Durand, L.: On sampling from a log-concave density using kinetic Langevin diffusions. Bernoulli 26(3), 1956–1988, 2020

    Article  MathSciNet  MATH  Google Scholar 

  17. Dolbeault, J., Klar, A., Mouhot, C., Schmeiser, C.: Exponential rate of convergence to equilibrium for a model describing fiber lay-down processes. Appl. Math. Res. eXpress 2013(2), 165–175, 2013

    MathSciNet  MATH  Google Scholar 

  18. Dolbeault, J., Mouhot, C., Schmeiser, C.: Hypocoercivity for kinetic equations with linear relaxation terms. Comptes Rendus Mathematique 347(9), 511–516, 2009

    Article  MathSciNet  MATH  Google Scholar 

  19. Dolbeault, J., Mouhot, C., Schmeiser, C.: Hypocoercivity for linear kinetic equations conserving mass. Trans. Am. Math. Soc. 367(6), 3807–3828, 2015

    Article  MathSciNet  MATH  Google Scholar 

  20. Eberle, A., Guillin, A., Zimmer, R.: Couplings and quantitative contraction rates for Langevin dynamics. Ann. Probab. 47(4), 1982–2010, 2019

    Article  MathSciNet  MATH  Google Scholar 

  21. Eckmann, J.-P., Hairer, M.: Spectral properties of hypoelliptic operators. Commun. Math. Phys. 235(2), 233–253, 2003

    Article  ADS  MathSciNet  MATH  Google Scholar 

  22. Evans, L.C.: Partial Differential Equations, vol. 19, American Mathematical Society, Philadelphia, 2010

  23. Galdi, G.: An Introduction to the Mathematical Theory of the Navier–Stokes Equations: Steady-State Problems, Springer, Berlin, 2011

  24. Gigli, N.: Answer to “Elliptic regularity with Gibbs measure satisfying Bakry–Emery condition”, MathOverflow. https://mathoverflow.net/q/335608 (version: 2019-07-06)

  25. Grothaus, M., Stilgenbauer, P.: Hypocoercivity for Kolmogorov backward evolution equations and applications. J. Funct. Anal. 267(10), 3515–3556, 2014

    Article  MathSciNet  MATH  Google Scholar 

  26. Grothaus, M., Stilgenbauer, P.: Hilbert space hypocoercivity for the Langevin dynamics revisited. Methods Funct. Anal. Topol. 22(02), 152–168, 2016

    MathSciNet  MATH  Google Scholar 

  27. Helffer, B., Nier, F.: Hypoelliptic Estimates and Spectral Theory for Fokker–Planck Operators and Witten Laplacians, vol. 1862, Springer, 2005

  28. Hérau, F.: Hypocoercivity and exponential time decay for the linear inhomogeneous relaxation Boltzmann equation. Asympt. Anal. 46(3–4), 349–359, 2006

    MathSciNet  MATH  Google Scholar 

  29. Hérau, F., Nier, F.: Isotropic hypoellipticity and trend to equilibrium for the Fokker–Planck equation with a high-degree potential. Arch. Ration. Mech. Anal. 171(2), 151–218, 2004

    Article  MathSciNet  MATH  Google Scholar 

  30. Herzog, D.P., Mattingly, J.C.: Ergodicity and Lyapunov functions for Langevin dynamics with singular potentials. Commun. Pure Appl. Math. 72(10), 2231–2255, 2019

    Article  MathSciNet  MATH  Google Scholar 

  31. Hooton, J.G.: Compact Sobolev imbeddings on finite measure spaces. J. Math. Anal. Appl. 83, 570–581, 1981

    Article  MathSciNet  MATH  Google Scholar 

  32. Hörmander, L.: Hypoelliptic second order differential equations. Acta Math. 119, 147–171, 1967

    Article  MathSciNet  MATH  Google Scholar 

  33. Iacobucci, A., Olla, S., Stoltz, G.: Convergence rates for nonequilibrium Langevin dynamics. Annales mathématiques du Québec 43(1), 73–98, 2019

    Article  MathSciNet  MATH  Google Scholar 

  34. Kolmogorov, A.: Zufallige bewegungen (zur theorie der Brownschen bewegung). Ann. Math. 66, 116–117, 1934

    Article  MathSciNet  MATH  Google Scholar 

  35. Kozlov, S.M.: Effective diffusion in the Fokker–Planck equation. Math. Notes Acad. Sci. USSR 45(5), 360–368, 1989

    MATH  Google Scholar 

  36. Ledoux, M.: A simple analytic proof of an inequality by P. Buser. Proc. Am. Math. Soc. 121(3), 951–959, 1994

    Article  MathSciNet  MATH  Google Scholar 

  37. Ledoux, M.: Spectral gap, logarithmic Sobolev constant, and geometric bounds. Surv. Differ. Geom. 9(1), 219–240, 2004

    Article  MathSciNet  MATH  Google Scholar 

  38. Leimkuhler, B., Sachs, M., Stoltz, G.: Hypocoercivity properties of adaptive Langevin dynamics. SIAM J. Appl. Math. 80(3), 1197–1222, 2020

    Article  MathSciNet  MATH  Google Scholar 

  39. Lu, Y., Mattingly, J.C.: Geometric ergodicity of Langevin dynamics with Coulomb interactions. Nonlinearity 33(2), 675, 2019

    Article  ADS  MathSciNet  MATH  Google Scholar 

  40. Ma, Y.-A., Chatterji, N.S., Cheng, X., Flammarion, N., Bartlett, P.L., Jordan, M.I.: Is there an analog of Nesterov acceleration for gradient-based MCMC? Bernoulli 27(3), 1942–1992, 2021

  41. Mattingly, J.C., Stuart, A.M., Higham, D.J.: Ergodicity for SDEs and approximations: locally Lipschitz vector fields and degenerate noise. Stoch. Process. Appl. 101(2), 185–232, 2002

    Article  MathSciNet  MATH  Google Scholar 

  42. Metafune, G., Pallara, D., Priola, E.: Spectrum of Ornstein–Uhlenbeck operators in lp spaces with respect to invariant measures. J. Funct. Anal. 196(1), 40–60, 2002

    Article  MathSciNet  MATH  Google Scholar 

  43. Mouhot, C., Neumann, L.: Quantitative perturbative study of convergence to equilibrium for collisional kinetic models in the torus. Nonlinearity 19(4), 969, 2006

  44. Otto, F., Villani, C.: Generalization of an inequality by Talagrand and links with the logarithmic Sobolev inequality. J. Funct. Anal. 173(2), 361–400, 2000

    Article  MathSciNet  MATH  Google Scholar 

  45. Pavliotis, G.A.: Stochastic Processes and Applications: Diffusion Processes, the Fokker–Planck and Langevin Equations, vol. 60, Springer, 2014

  46. Risken, H.: Fokker–Planck Equation: Methods of Solution and Applications, Springer Series in Synergetics, 1989

  47. Roussel, J., Stoltz, G.: Spectral methods for Langevin dynamics and associated error estimates. ESAIM Math. Model. Numer. Anal. 52(3), 1051–1083, 2018

  48. Sason, I., Verdú, S.: \(f\)-divergence inequalities. IEEE Trans. Inf. Theory 62(11), 5973–6006, 2016

    Article  MathSciNet  MATH  Google Scholar 

  49. Stoltz, G., Trstanova, Z.: Langevin dynamics with general kinetic energies. Multiscale Model. Simul. 16(2), 777–806, 2018

    Article  MathSciNet  MATH  Google Scholar 

  50. Stoltz, G., Vanden-Eijnden, E.: Longtime convergence of the temperature-accelerated molecular dynamics method. Nonlinearity 31(8), 3748–3769, 2018

    Article  ADS  MathSciNet  MATH  Google Scholar 

  51. Talay, D.: Stochastic Hamiltonian systems: exponential convergence to the invariant measure, and discretization by the implicit Euler scheme. Markov Process. Relat. Fields 8(2), 163–198, 2002

    MathSciNet  MATH  Google Scholar 

  52. Tropper, M.M.: Ergodic and quasideterministic properties of finite-dimensional stochastic systems. J. Stat. Phys. 17(6), 491–509, 1977

    Article  ADS  MathSciNet  MATH  Google Scholar 

  53. Villani, C.: Hypocoercive diffusion operators. Bollettino dell’Unione Matematica Italiana 10-B(2), 257–275, 2007

  54. Villani, C.: Hypocoercivity. Mem. Am. Math. Soc. 202(950), 66, 2009

  55. Wu, L.: Large and moderate deviations and exponential convergence for stochastic damping Hamiltonian systems. Stoch. Process. Appl. 91(2), 205–238, 2001

    Article  MathSciNet  MATH  Google Scholar 

Download references

Acknowledgements

This research is supported in part by National Science Foundation via Grants DMS-1454939 and CCF-1910571. We would like to thank Rong Ge, Yulong Lu, Jonathan Mattingly, Jean-Christophe Mourrat, and Gabriel Stoltz for helpful discussions, and thank Felix Otto for discussions and providing an idea leading to the proof of Lemma 2.6. LW would also like to thank Scott Armstrong [3] and Nicola Gigli [24] for answering our question on MathOverflow, which lead to our proof of Lemma 2.4 (ii).

Author information

Authors and Affiliations

Authors

Corresponding author

Correspondence to Lihan Wang.

Ethics declarations

Conflict of interest Statement

The authors have no conflict of interest.

Additional information

Communicated by C. Le Bris.

Publisher's Note

Springer Nature remains neutral with regard to jurisdictional claims in published maps and institutional affiliations.

Appendices

Appendix A. The Decay Rate for Isotropic Quadratic Potential

For isotropic quadratic potential, an explicit expression for the spectral gap of \({\mathcal {L}}\) is available (thus also the decay rate in (7)). Note that while the result is stated for \(d = 1\), it trivially extends to arbitrary dimension for isotropic quadratic potential as different coordinates are independent. The spectrum is also explicitly known for \(V=0\) and \(x\in {\mathbb {T}}^d\) on a torus, see [35].

Theorem 3

([46, (10.83)], [42, Theorem 3.1]) When \(U(x) = \frac{m}{2}|x|^2\), \(d = 1\), the spectrum of the operator \(-{\mathcal {L}}\) is given by

$$\begin{aligned} \left\{ \lambda _{i,j} {:}{=} \frac{\gamma }{2} (i + j) + \frac{\sqrt{\gamma ^2 - 4 m}}{2}(i-j), \qquad i,j = 0, 1, 2, \cdots .\right\} . \end{aligned}$$

Let \(\lambda _{\text {exact}}\) be the spectral gap for the real component of \(\{\lambda _{i,j}\}_{i,j\geqslant 0}\). Notice that the spectral gap is always achieved when \(i = 0\) and \(j = 1\), thus

$$\begin{aligned} \lambda _{\text {exact}} = \mathfrak {Re}\left( \frac{\gamma }{2} - \frac{\sqrt{\gamma ^2-4m}}{2}\right) . \end{aligned}$$
(45)

Corollary A.1

For any dimension d, for isotropic potential \(U(x) = \frac{m}{2}|x|^2\), (7) holds with the decay rate \(\lambda _{\text {exact}}\).

Appendix B. The DMS Hypocoercive Estimation

In this section, we will revisit the decay rate by DMS estimation [18, 19], adapted and summarized for underdamped Langevin equation in [47, Sec. 2]. In the first part of this section, we will review the main result based on [47]; in addition, we will provide a new estimate of the operator norm of \( \left\Vert {\mathcal {A}}{\mathcal {L}}_{\text {ham}}(1-\Pi _v)\right\Vert _{L^2(\rho _{\infty }) \rightarrow L^2(\rho _{\infty })}\), which leads into a more explicit expression of the decay rate. In the second part, we will present the asymptotic analysis of the decay rate with respect to m and \(\gamma \), under the assumption that \(\nabla _x^2 U \geqslant -2\, \textrm{Id}\).

1.1 Revisiting the DMS Hypocoercive Estimation in \(L^2(\rho _{\infty })\)

Let us first define an operator

$$\begin{aligned} {\mathcal {A}}= \left( 1 + ({\mathcal {L}}_{\text {ham}}\Pi _v)^* ({\mathcal {L}}_{\text {ham}}\Pi _v)\right) ^{-1} ({\mathcal {L}}_{\text {ham}}\Pi _v)^* \end{aligned}$$
(46)

and a Lyapunov function \(\textsf{E}\) for \(\phi (x, v)\) by

$$\begin{aligned} \textsf{E}(\phi ) = \frac{1}{2} \left\Vert \phi \right\Vert ^2_{L^2(\rho _{\infty })} -\epsilon \left( {\mathcal {A}}\phi , \phi \right) _{L^2(\rho _{\infty })}, \end{aligned}$$
(47)

where \(\epsilon \in (-1, 1)\) is some quantity depending on \({\mathcal {L}}\), to be specified below. The functional \(\textsf{E}\) is equivalent to \(L^2(\rho _{\infty })\) norm in the following sense (see e.g., [47, Eq. (17)]),

$$\begin{aligned} \frac{1- |\epsilon |}{2} \left\Vert \phi \right\Vert _{L^2(\rho _{\infty })}^2 \leqslant \textsf{E}(\phi ) \leqslant \frac{1+|\epsilon |}{2} \left\Vert \phi \right\Vert _{L^2(\rho _{\infty })}^2. \end{aligned}$$
(48)

Theorem 4

(See [47, Theorem 1]) Assume that the Poincaré inequality (10) holds and there exists \(\textsf{R}_{\text {ham}}< \infty \) such that

$$\begin{aligned} \left\Vert {\mathcal {A}}{\mathcal {L}}_{\text {ham}}(1-\Pi _v)\right\Vert _{L^2(\rho _{\infty }) \rightarrow L^2(\rho _{\infty })} \leqslant \textsf{R}_{\text {ham}}. \end{aligned}$$
(49)

Suppose \(\epsilon \in (-1, 1)\) is chosen such that \(\lambda _{\text {DMS}} = \lambda _{\text {DMS}} (\gamma , m, \textsf{R}_{\text {ham}}, \epsilon ) > 0\), where

$$\begin{aligned} \lambda _{\text {DMS}} {:}{=} \frac{\gamma - \frac{\epsilon }{1 + m} - \sqrt{\epsilon ^2 (\textsf{R}_{\text {ham}}+ \frac{\gamma }{2})^2 + \left( \gamma - \frac{2 m + 1}{m+1}\epsilon \right) ^2}}{2(1+|\epsilon |)}. \end{aligned}$$
(50)

Then for any solution f(txv) of (4) with \(\int f_0\ \,\textrm{d}\rho _{\infty }= 0\), we have

$$\begin{aligned} \left\Vert f(t, \cdot , \cdot )\right\Vert _{L^2(\rho _{\infty })} \leqslant \sqrt{\frac{1+|\epsilon |}{1-|\epsilon |}} \left\Vert f_0\right\Vert _{L^2(\rho _{\infty })} e^{- \lambda _{\text {DMS}}\ t}. \end{aligned}$$

Notice that when \(\epsilon = 0\), the rate \(\lambda _{\text {DMS}} = 0\), which reduces to the conclusion that \(\left\Vert f(t, \cdot , \cdot )\right\Vert _{L^2(\rho _{\infty })}\) is non-increasing in time t. The existence of \(\textsf{R}_{\text {ham}}\) has been studied under fairly general assumptions on the potential U(x) in [19, Sec. 2]. In the Proposition B.1 below, we provide a simpler estimation of \(\textsf{R}_{\text {ham}}\) only under the assumption of lower bound on Hessian; see the Appendix B.3 for its proof. The first part of the proof is the same as [19, Lemma 4]; the simplicity in our approach comes from the application of Bochner’s formula. It is interesting to observe that \(\textsf{R}_{\text {ham}}\) does not depend on m when U is an isotropic quadratic potential.

Proposition B.1

Assume there exists \(K \in {\mathbb {R}}\) such that \(\nabla _x^2 U \geqslant -K\, \textrm{Id}\) for all \(x \in {\mathbb {R}}^d\), then we can choose

$$\begin{aligned} \textsf{R}_{\text {ham}}= \sqrt{\max \{K, 2\}}. \end{aligned}$$
(51)

such that (49) is satisfied.

For the isotropic case \(U(x) = \frac{m}{2}|x|^2\), we have

$$\begin{aligned} \left\Vert {\mathcal {A}}{\mathcal {L}}_{\text {ham}}(1-\Pi _v)\right\Vert _{L^2(\rho _{\infty }) \rightarrow L^2(\rho _{\infty })} = \sqrt{2}. \end{aligned}$$

Thus the optimal choice of \(\textsf{R}_{\text {ham}}\) is \(\sqrt{2}\) and (51) is tight in this case.

As an immediate consequence, if it holds that \(\nabla _x^2 U \geqslant -2\, \textrm{Id}\), we can take \( \textsf{R}_{\text {ham}}= \sqrt{2}\), which is tight for the isotropic case.

1.2 Asymptotic Analysis of the Decay Rate

In this subsection, we shall assume that \(\nabla _x^2 U \geqslant -2\, \textrm{Id}\), thus we can choose \(\textsf{R}_{\text {ham}}= \sqrt{2}\), according to the Proposition B.1. To remove the dependence on the parameter \(\epsilon \) and to find the optimal decay rate, let us introduce

$$\begin{aligned} \begin{aligned} \Lambda _{\text {DMS}}(\gamma , m)&{:}{=} \sup _{\epsilon \in (-1, 1)}\ \lambda _{\text {DMS}}(\gamma , m, \sqrt{2}, \epsilon )\\&= \sup _{\epsilon \in (-1, 1)} \frac{\gamma - \frac{\epsilon }{1 + m} - \sqrt{\epsilon ^2 (\sqrt{2} + \frac{\gamma }{2})^2 + \left( \gamma - \frac{2 m + 1}{m+1}\epsilon \right) ^2}}{2(1+|\epsilon |)}, \end{aligned} \end{aligned}$$
(52)

provided that the supremum is not achieved at the boundary i.e., \(\epsilon = 1^{-}\) or \(\epsilon = (-1)^{+}\). Observe that

  • When \(\epsilon = 0\), \(\lambda _{\text {DMS}}(\gamma , m, \sqrt{2}, 0) = 0\);

  • When \(\epsilon = (-1)^{+}\), \(\lambda _{\text {DMS}}(\gamma , m, \sqrt{2}, (-1)^{+}) < 0\).

Therefore, the supremum can only be achieved at \(\epsilon = 1^{-}\), or the critical points of the expression on the right hand side of (52). In general, it is hard to obtain a simple explicit expression of \(\Lambda _{\text {DMS}}(\gamma , m)\). Therefore, we shall consider the following asymptotic regions:

Proposition B.2

  1. (i)

    For fixed \(m = O(1)\), we have

    $$\begin{aligned} \Lambda _{\text {DMS}}(\gamma , m) = \left\{ \begin{aligned} \left( \frac{-(1+m) \sqrt{3m^2+4m+1} + 3m^2+3m+1}{6m^2+8m+3}\right) \gamma +O(\gamma ^2),&\qquad \text{ when } \ \gamma \rightarrow 0;\\ \frac{4m^2}{(1+m)^2 }\gamma ^{-1} + O(\gamma ^{-2}),&\qquad \text{ when } \ \gamma \rightarrow \infty .\\ \end{aligned} \right. \end{aligned}$$
    (53)
  2. (ii)

    Consider coupled asymptotic regime \(\gamma = b\sqrt{m}\) (or equivalently \(m =\left( \gamma /b\right) ^2\)) for some \(b = O(1)\), we have

    $$\begin{aligned} \Lambda _{\text {DMS}}(\gamma , m) = \left\{ \begin{aligned}&\frac{\gamma ^5}{2b^4} + O(\gamma ^6),&\qquad \text { when } \gamma \rightarrow 0;\\ \frac{4}{\gamma } + O(\gamma ^{-2}),&\qquad \text { when } \gamma \rightarrow \infty .\\ \end{aligned}\right. \end{aligned}$$
    (54)

The proof can be found in Appendix B.3. The scaling in the first case is already known in e.g., [17, 26, 47]; in the above proposition, we simply explicitly calculate the leading order term. The second case is relevant when we choose \(\gamma \) to optimize the convergence rate according to m and for the regime \(m\rightarrow 0\).

1.3 Proofs of the Propositions in Appendix

Proof of Proposition B.1

We first consider the case that Hessian is bounded from below. It is equivalent to consider the operator norm of

$$\begin{aligned} -(1-\Pi _v) {\mathcal {L}}_{\text {ham}}{\mathcal {A}}^* = -(1-\Pi _v) {\mathcal {L}}_{\text {ham}}^2 \Pi _v \left( 1 + ({\mathcal {L}}_{\text {ham}}\Pi _v)^* ({\mathcal {L}}_{\text {ham}}\Pi _v)\right) ^{-1}. \end{aligned}$$

Notice that this operator is supported on \(\text {Ran}(\Pi _v)\) from the observation that \({\mathcal {A}}= \Pi _v {\mathcal {A}}\), it is then equivalent to find the smallest \(\textsf{R}_{\text {ham}}\) such that for any \(\phi (x,v)\) with \(\Pi _v \phi = \phi \) (i.e., \(\phi (x,v) \equiv \phi (x)\) is a function of x only), we have

$$\begin{aligned} \left\Vert -(1-\Pi _v) {\mathcal {L}}_{\text {ham}}{\mathcal {A}}^* \phi \right\Vert _{L^2(\rho _{\infty })}\leqslant \textsf{R}_{\text {ham}}\left\Vert \phi \right\Vert _{L^2(\rho _{\infty })} = \textsf{R}_{\text {ham}}\left\Vert \phi \right\Vert _{L^2(\mu )}. \end{aligned}$$
(55)

Given such a function \(\phi \) with \(\Pi _v \phi = \phi \), define

$$\begin{aligned}\varphi {:}{=} \left( 1 + ({\mathcal {L}}_{\text {ham}}\Pi _v)^* ({\mathcal {L}}_{\text {ham}}\Pi _v)\right) ^{-1} \phi .\end{aligned}$$

It is easy to check that \(\Pi _v \varphi = \varphi \). By simplifying the above equation with (5) and (9),

$$\begin{aligned} \phi (x) = \varphi (x) - \Delta _x \varphi (x) + \nabla _x U(x) \cdot \nabla _x \varphi = \varphi (x) + \nabla _x^* \nabla _x \varphi (x). \end{aligned}$$
(56)

Furthermore, by some straightforward calculation, we have

$$\begin{aligned} -(1-\Pi _v) {\mathcal {L}}_{\text {ham}}{\mathcal {A}}^* \phi = - (1-\Pi _v) {\mathcal {L}}_{\text {ham}}^2 \Pi _v \varphi = - \sum _{i,j} (v_i v_j - \delta _{i,j}) \partial _{x_i, x_j} \varphi . \end{aligned}$$

Thus

$$\begin{aligned} \left\Vert -(1-\Pi _v) {\mathcal {L}}_{\text {ham}}{\mathcal {A}}^* \phi \right\Vert _{L^2(\rho _{\infty })}^2&= \int \left( \sum _{i,j} (v_i v_j - \delta _{i,j}) \partial _{x_i, x_j} \varphi \right) ^2\,\textrm{d}\rho _{\infty }\\&= 2 \sum _{i,j} \int \left( \partial _{x_i, x_j} {\varphi }\right) ^2\,\textrm{d}\mu . \end{aligned}$$

Then, by Bochner’s formula,

$$\begin{aligned} \Vert -(1-\Pi _v) {\mathcal {L}}_{\text {ham}}&{\mathcal {A}}^* (\phi ) \Vert _{L^2(\rho _{\infty })}^2 \\ =&\ 2 \int \nabla _x \varphi \cdot \nabla _x \nabla _x^* \nabla _x {\varphi } - \nabla _x {\varphi } \cdot \nabla _x^2 U \nabla _x {\varphi } - \nabla _x^* \nabla _x \left( \frac{|\nabla _x {\varphi }|^2}{2}\right) \,\textrm{d}\mu \\ =&\ 2 \int |\nabla _x^*\nabla _x {\varphi }|^2 - \nabla _x {\varphi } \cdot \nabla _x^2 U \nabla _x {\varphi }\,\textrm{d}\mu \\ \leqslant&\ 2 \left( \int |\nabla _x^*\nabla _x {\varphi }|^2\,\textrm{d}\mu + K \int |\nabla _x {\varphi }|^2\,\textrm{d}\mu \right) \\ \leqslant&\ \max \left\{ K, 2\right\} \left( \int |\nabla _x^* \nabla _x {\varphi }|^2\,\textrm{d}\mu + 2 \int |\nabla _x {\varphi }|^2\,\textrm{d}\mu \right) . \end{aligned}$$

From (56), we have

$$\begin{aligned} \left\Vert {\phi }\right\Vert _{L^2(\mu )}^2&= \int {\varphi }^2 + 2 {\varphi }\ \nabla _x^* \nabla _x {\varphi } + \left|\nabla _x^*\nabla _x {\varphi }\right|^2\ \textrm{d}\mu \\&\geqslant 2 \int |\nabla _x{\varphi }|^2\,\textrm{d}\mu + \int |\nabla _x^* \nabla _x {\varphi }|^2\,\textrm{d}\mu . \end{aligned}$$

By combining the last two equations,

$$\begin{aligned} \left\Vert -(1-\Pi _v) {\mathcal {L}}_{\text {ham}}{\mathcal {A}}^* (\phi )\right\Vert _{L^2(\rho _{\infty })}^2&\leqslant \max \{K, 2\} \left\Vert {\phi }\right\Vert _{L^2(\mu )}^2, \end{aligned}$$

which yields (51).

We now consider the isotropic case. Recall that the operator norm of \({\mathcal {A}}{\mathcal {L}}_{\text {ham}}(1-\Pi _v)\) is the smallest \(\textsf{R}_{\text {ham}}\) such that (55) holds. Let us consider the elliptic PDE (56). By the choice \(U(x) = \frac{m}{2}|x|^2\),

$$\begin{aligned} {\phi }(x) = \left( 1 + m (x - \frac{1}{m} \nabla _x) \cdot \nabla _x \right) {\varphi }(x). \end{aligned}$$

Then by rescaling the variable \(x = \frac{y}{\sqrt{m}}\) and rescaling the functions \(\bar{\phi }(y) {:}{=} {\phi }(x) = \phi \left( \frac{y}{\sqrt{m}}\right) \), \(\bar{\varphi }(y) {:}{=} {\varphi }(x) = {\varphi }\left( \frac{y}{\sqrt{m}}\right) \), we have

$$\begin{aligned} \bar{\phi }(y) = \left( 1 + m (y - \nabla _y) \cdot \nabla _y\right) \bar{\varphi }(y). \end{aligned}$$
(57)

In addition, by rewriting (55), we need to find the smallest \(\textsf{R}_{\text {ham}}\) such that

$$\begin{aligned} 2 m^2 \sum _{i,j} \int \left|\partial _{y_i, y_j} \bar{\varphi }(y)\right|^2\ e^{-\frac{|y|^2}{2}}\,\textrm{d}y\leqslant \textsf{R}_{\text {ham}}^2 \int \bar{\phi }(y)^2 e^{-\frac{|y|^2}{2}}\,\textrm{d}y. \end{aligned}$$
(58)

Next, let us expand the last equation by probabilists’ Hermite polynomials \(H_k(z) {:}{=} (z - \frac{\textrm{d}}{\textrm{d}z})^k \cdot 1\) for integers \(k\geqslant 0\). Recall two important properties

$$\begin{aligned} H'_k(z) = k H_{k-1}(z), \qquad \frac{1}{\sqrt{2\pi }}\int H_j(z) H_k(z) e^{-\frac{z^2}{2}}\,\textrm{d}z = k!\, \delta _{j,k}. \end{aligned}$$

Given \(\varvec{n} = (n_1, n_2, \cdots , n_d)\), define

$$\begin{aligned} H_{\varvec{n}}(y) {:}{=} H_{n_1}(y_1) H_{n_2}(y_2) \cdots H_{n_d}(y_d). \end{aligned}$$

By the above properties, it is easy to show that if \(\bar{\varphi } = H_{\varvec{n}}\), then \(\bar{\phi } = N_{\varvec{n}} H_{\varvec{n}}\), where \(N_{\varvec{n}} {:}{=} 1 + m \sum _{i} n_i\). Thus if \(\bar{\varphi }(y) = \sum _{\varvec{n}} a_{\varvec{n}}H_{\varvec{n}}\), then we have \(\bar{\phi } = \sum _{n} a_{\varvec{n}} N_{\varvec{n}} H_{\varvec{n}}\). By such an expansion, (58) can be rewritten as

$$\begin{aligned} 2m^2 \sum _{i, j} \sum _{\varvec{n}} a_{\varvec{n}}^2 (n_i n_j - \delta _{i,j} n_i) \prod _{k=1}^{d} n_k! \leqslant \textsf{R}_{\text {ham}}^2 \sum _{\varvec{n}} a_{\varvec{n}}^2 N_{\varvec{n}}^2 \prod _{k=1}^{d} n_k! \end{aligned}$$

Then finding the operator norm of \({\mathcal {A}}{\mathcal {L}}_{\text {ham}}(1-\Pi _v)\) is equivalent to finding the smallest \(\textsf{R}_{\text {ham}}\) such that for any \(\varvec{n}\), one has

$$\begin{aligned} \sum _{i,j} (n_i n_j - \delta _{i,j} n_i) \leqslant \frac{\textsf{R}_{\text {ham}}^2}{2 m^2 } N_{\varvec{n}}^2 \equiv \frac{\textsf{R}_{\text {ham}}^2}{2 m^2 } \left( 1 + m \sum _i n_i\right) ^2. \end{aligned}$$

When \(n_1 \rightarrow \infty \) and \(n_2, n_3, \cdots , n_d = 0\), we know that \(\frac{\textsf{R}_{\text {ham}}^2}{2} \geqslant 1\). Also observe that

$$\begin{aligned} \sum _{i,j} (n_i n_j - \delta _{i,j} n_i) \leqslant \left( \sum _{i} n_i\right) ^2 = \frac{1}{m^2} \left( m \sum _i n_i\right) ^2\leqslant \frac{1}{m^2} \left( 1 + m \sum _i n_i\right) ^2. \end{aligned}$$

Therefore, \(\frac{\textsf{R}_{\text {ham}}^2}{2} = 1\) is sufficient.

In summary, \(\left\Vert {\mathcal {A}}{\mathcal {L}}_{\text {ham}}(1-\Pi _v)\right\Vert _{L^2(\rho _{\infty }) \rightarrow L^2(\rho _{\infty })} = \sqrt{2}\) and the optimal choice of \(\textsf{R}_{\text {ham}}\) is \(\sqrt{2}\). \(\quad \square \)

Proof of Proposition B.2

We used Maple software to help verify the asymptotic expansion.

Part (i): \(m = O(1)\).

  • (when \(\gamma \rightarrow 0\)). Via asymptotic expansion, we have

    $$\begin{aligned} \lambda _{\text {DMS}}(\gamma , m, \sqrt{2}, 1^{-}) = - \frac{1+\sqrt{6 m^2 + 8m + 3}}{4(1+m)} + O(\gamma ) < 0. \end{aligned}$$

    Thus the supremum is not obtained at \(\epsilon = 1^{-}\). Then let us consider critical points within the domain \((-1, 1)\), whose asymptotic expansions are

    $$\begin{aligned} \epsilon _{\pm } = \frac{(6m^2+5m+1 \pm \sqrt{3m^2+4m+1})(1+m)}{18m^3+30m^2+17m+3} \gamma + O(\gamma ^2) > 0. \end{aligned}$$

    After comparison, the larger decay rate is obtained at \(\epsilon _{-}\) with the value in (53).

  • (when \(\gamma \rightarrow \infty \)). Similarly, via asymptotic expansion, we have

    $$\begin{aligned} \lambda _{\text {DMS}}(\gamma , m, \sqrt{2}, 1^{-}) = -\frac{\frac{\sqrt{5}}{2}-1}{4}\gamma + O(1) < 0. \end{aligned}$$

    Thus we need to consider the critical points. It turns out, there is only one critical point within the domain \((-1, 1)\), which is \(\varepsilon = \frac{8m}{1+m}\gamma ^{-1} + O(\gamma ^{-2})\) with the decay rate in (53).

Part (ii): \(\gamma = b\sqrt{m}\) with \(b=O(1)\).

  • (when \(\gamma \rightarrow 0\)). Via asymptotic expansion, one could check that

    $$\begin{aligned} \lambda _{\text {DMS}}(\gamma , m=(\gamma /b)^2, \sqrt{2}, 1^{-}) = -\frac{1+\sqrt{3}}{4} + O(\gamma ) < 0. \end{aligned}$$

    Thus, we only need to consider the decay rate at critical points, which are given by

    $$\begin{aligned} \epsilon _1 = \frac{\gamma ^3}{b^2} + O(\gamma ^4), \qquad \epsilon _2 = \frac{2}{3} \gamma + O(\gamma ^2). \end{aligned}$$

    and the associated decay rates are

    $$\begin{aligned} \lambda _{\text {DMS}}(\gamma , m=(\gamma /b)^2, \sqrt{2}, \epsilon _1)&= \frac{\gamma ^5}{2b^4} + O(\gamma ^{6})> 0;\\ \lambda _{\text {DMS}}(\gamma , m=(\gamma /b)^2, \sqrt{2}, \epsilon _2)&= -\frac{1}{3} \gamma + O(\gamma ^2) < 0. \end{aligned}$$

    Therefore, the optimal decay rate is obtained at \(\epsilon _1\), which gives (54).

  • (when \(\gamma \rightarrow \infty \)). Via asymptotic expansion, one could obtain

    $$\begin{aligned} \lambda _{\text {DMS}}(\gamma , m=(\gamma /b)^2, \sqrt{2}, 1^{-}) = -\frac{\sqrt{5}-2}{8}\gamma + O(1) < 0. \end{aligned}$$

    Thus the supremum in (52) cannot be obtained at \(\epsilon = 1^{-}\). Then, let us look at the critical points. It turns out there is only one within the interval \((-1, 1)\), which is \(\epsilon _1 = \frac{8}{\gamma } + O(\gamma ^{-2})\). The optimal decay rate must be achieved at \(\epsilon _1\), with the expression given in (54). \(\quad \square \)

Rights and permissions

Springer Nature or its licensor (e.g. a society or other partner) holds exclusive rights to this article under a publishing agreement with the author(s) or other rightsholder(s); author self-archiving of the accepted manuscript version of this article is solely governed by the terms of such publishing agreement and applicable law.

Reprints and permissions

About this article

Check for updates. Verify currency and authenticity via CrossMark

Cite this article

Cao, Y., Lu, J. & Wang, L. On Explicit \(L^2\)-Convergence Rate Estimate for Underdamped Langevin Dynamics. Arch Rational Mech Anal 247, 90 (2023). https://doi.org/10.1007/s00205-023-01922-4

Download citation

  • Received:

  • Accepted:

  • Published:

  • DOI: https://doi.org/10.1007/s00205-023-01922-4

Navigation